Probability - Random Variables

Click For Summary
The discussion revolves around a homework problem involving a test of 10 true-false questions. The first part asks for the total number of ways to complete the test, which is calculated as 2^10, equating to 1024 ways. The second part involves a student answering questions by flipping a coin, leading to a binomial distribution for the number of correct answers, denoted as X. The probability of answering at most 5 questions correctly is calculated using the binomial cumulative distribution function, specifically binomcdf(10, 0.5, 5). Participants confirm the correctness of these calculations and express some uncertainty about the binomial function format.
tjackson
Messages
4
Reaction score
0

Homework Statement



1. A test consists of 10 true-false questions.
(a) In how many ways can it be completed? (HINT: The task of completing the test consists
of 10 stages. Use the Product Rule.)

(b) A student answers the questions by
flipping a coin. Let X denote the number of correctly
answered questions.
(i) The distribution of X is:

binomial
hypergeometric
negative binomial
Poisson

(ii) Find the probability he/she will answer correctly at most 5 of the questions.





Homework Equations

/ Attempt at a solution

a) i believe this is just 210?

b.) i.) is this binomial??

ii.) \stackrel{n}{k} * pk * (1-p)(n-k)

p = 1/2
k = 5
n = 10

\stackrel{n}{k}
is n choose k
 
Physics news on Phys.org
a)correct
b)binomial
binomcdf(10,.5,5)

It's been awhile since I took statistics. There is a chance I'm wrong.
 
FileDeleted said:
a)correct
b)binomial
binomcdf(10,.5,5)

It's been awhile since I took statistics. There is a chance I'm wrong.

Your answers are correct.

RGV
 
So these statements are correct?

p = 1/2
k = 5
n = 10

I apologize, I am not familiar with the 'binomcdf(10,.5,5)' format
 
Question: A clock's minute hand has length 4 and its hour hand has length 3. What is the distance between the tips at the moment when it is increasing most rapidly?(Putnam Exam Question) Answer: Making assumption that both the hands moves at constant angular velocities, the answer is ## \sqrt{7} .## But don't you think this assumption is somewhat doubtful and wrong?

Similar threads

  • · Replies 8 ·
Replies
8
Views
1K
Replies
2
Views
1K
  • · Replies 2 ·
Replies
2
Views
1K
  • · Replies 3 ·
Replies
3
Views
1K
  • · Replies 5 ·
Replies
5
Views
2K
Replies
1
Views
1K
  • · Replies 4 ·
Replies
4
Views
2K
  • · Replies 1 ·
Replies
1
Views
1K
  • · Replies 2 ·
Replies
2
Views
1K
  • · Replies 2 ·
Replies
2
Views
2K